Skip to content
Snippets Groups Projects
Commit 5a35043c authored by Kroum Tzanev's avatar Kroum Tzanev
Browse files

rajout du DS2 du 18 avril 2024

parent b0216298
Branches
No related tags found
No related merge requests found
Pipeline #56162 passed
File added
File added
\documentclass[a4paper,12pt,reqno]{amsart}
\usepackage{lille}
\lilleset{
% solutions,
% sans enonces,
titre=\sisujet{Examen final}\sisolutions{Solutions de l'examen},
date=18 avril 2024,
duree=3 heures,
}
\input{m61proba}
\begin{document}
\sisujet{
\tsvp
\vspace{7mm}
\attention~
\emph{Les documents et les objets électroniques sont interdits. Les exercices sont indépendants. Toutes les réponses doivent être justifiées.}
\vspace{7mm}
}
% ----------------------------------------------- 1
\begin{exo}
Soient $X$ et $Y$ deux variables aléatoires indépendantes. On suppose que $X$ suit la loi géométrique de paramètre $\alpha$ et que $Y$ suit la loi géométrique de paramètre $\beta$, avec $\alpha,\beta\in ]0, 1[$. On note
\[
Z \coloneqq \min(X, Y).
\]
\begin{enumerate}
\item Pour tout $k\in\N$, calculer $\PP{X > k}$ et $\PP{Y > k}$.
\item Calculer $\PP{Z > k}$, puis en déduire la loi de $Z$. Identifier cette loi.
\end{enumerate}
\end{exo}
\begin{solution}
\begin{enumerate}
\item Pour tout $k\in\N$, on a
\[
\PP{X > k}
= \sum_{j > k} \PP{X = j}
= \sum_{j \geq k+1} \alpha(1-\alpha)^{j-1}
= \alpha(1-\alpha)^k\frac{1}{1-(1-\alpha)}
= (1-\alpha)^k,
\]
en utilisant la somme de la série géométrique de premier terme $\alpha(1-\alpha)^k$ et de raison $(1-\alpha)\in]0,1[$. De même, on trouve
\[
\PP{Y > k} = (1-\beta)^k.
\]
\item Pour tout $k\in\N$, on a
\[
\PP{Z > k}
= \PP{X > k,\, Y > k}
= \PP{X>k}\times\PP{Y > k}
= ((1-\alpha)(1-\beta))^k
\]
par indépendance de $X$ et $Y$. En posant $\gamma \coloneqq \alpha + \beta - \alpha\beta$ on trouve $\PP{Z > k} = (1-\gamma)^k$. Ainsi, pour tout $k\geq 1$,
\[
\PP{Z = k}
= \PP{Z > k-1} - \PP{Z > k}
= \gamma(1-\gamma)^{k-1}.
\]
Comme $Z = \min(X,Y)\geq 1$, vu que $X\geq 1$ et $Y\geq 1$, ceci signifie que $Z$ suit une loi géométrique de paramètre $\gamma$. On vérifie aussi que $\gamma\in ]0,1[$, puisque $\alpha,\beta\in ]0,1[$.
\end{enumerate}
\end{solution}
% ----------------------------------------------- 2
\begin{exo}
Soient $\mu \in \R$ et $\sigma \in \R^{*}$. Une variable aléatoire $X$ à valeurs dans $]0,+\infty[$ est dite suivre une loi \emph{log-normale} de paramètres $(\mu, \sigma^2)$ si $Y = \log X$ suit la loi gaussienne $\Nor{\mu,\sigma^2}$. On note $\LogNor{\mu,\sigma^2}$ la loi log-normale de paramètres $(\mu, \sigma^2)$.
\begin{enumerate}
\item Exprimer $X$ à l'aide d'une variable $Z$ (à préciser) qui suit une loi normale centrée réduite.
En déduire la fonction de répartition de $X$ à l'aide de la fonction $\Phi$ de répartition de $Z$.
Calculer ensuite explicitement la densité de $X$.
\item Calculer l'espérance de $X$.
\item Montrer que $X^r\sim\LogNor{r\mu,r^2\sigma^2}$ pour tout $r \neq 0$. En déduire la valeur de $\EE{X^r}$ pour tout $r\in\R$. Calculer la variance de $X$.
\item Calculer $\EE{e^{uX}}$ pour tout $u >0$.
\end{enumerate}
\end{exo}
\begin{solution}
\begin{enumerate}
\item\label{s2a} On a $X = e^Y$ avec $Y\sim\Nor{\mu, \sigma^2}$. En posant $Z\coloneqq \frac{Y-\mu}{\sigma}$ on trouve $X = e^{\mu +\sigma Z}$ avec $Z\sim\Nor{0, 1}$. Comme $X\geq0$, on en déduit $\PP{X\leq x} = 0$ pour tout $x\leq 0$, et pour tout $x > 0$ on a
\[
\PP{X \leq x}
= \PP{Z \leq \frac{\log x - \mu}{\sigma}}
= \Phi\pa{\frac{\log x - \mu}{\sigma}}.
\]
En dérivant on trouve que la densité de $X$ vaut
\[
\rho_X(x)
= \rho_Z\pa{\frac{\log x - \mu}{\sigma}}\frac{1}{\sigma x}
= \frac{1}{\sigma x \sqrt{2\pi}} \exp\left[ -\frac{(\log x -\mu)^2}{2\sigma^2}\right]
\]
sur $]0,\infty[$ et zéro ailleurs.
\item\label{s2b} En utilisant le théorème de transfert, on trouve
\[
\EE{X}
= \EE{e^{\mu +\sigma Z}}
= e^{\mu}\tfrac{1}{\sqrt{2\pi}}\int_{-\infty}^\infty e^{\sigma t} e^{-\frac{t^2}{2}} \dd t
= e^{\mu + \frac{\sigma^2}{2}}\tfrac{1}{\sqrt{2\pi}}\int_{-\infty}^\infty e^{-\frac{(t-\sigma)^2}{2}} \dd t
= e^{\mu + \frac{\sigma^2}{2}}.
\]
\item On a
\[
X^r = e^{r\mu + (r\sigma) Z}
\]
et ceci entraîne par \eqref{s2a} que $X^r\sim\LogNor{r\mu,r^2\sigma^2}$ pour tout $r \neq 0$, et donc par \eqref{s2b} que
\[
\EE{X^r} = e^{r\mu + r^2\sigma^2/2}
\]
pour tout $r\in\R$. On en déduit la variance de $X$
\[
\VV{X}
= \EE{X^2} - \bpa{\EE{X}}^2
= e^{2\mu + 2\sigma^2} - e^{2\mu + \sigma^2}
= e^{2\mu + \sigma^2} (e^{\sigma^2} -1).
\]
\item On a par le \eqref{s2a}, et en appliquant le théorème de transfert,
\[
\EE{e^{uX}}
= \EE{e^{ue^{\mu + \sigma Z}}}
= \frac{1}{\sqrt{2\pi}}\int_\R e^{ue^{\mu + \sigma x} - x^2/2} \dd x
= \infty
\]
puisque $u, \sigma > 0$ et donc $e^{ue^{\mu + \sigma x} - x^2/2}\to\infty$ quand $x\to\infty$.
\end{enumerate}
\end{solution}
% ----------------------------------------------- 3
\begin{exo}
\emph{Les trois questions dans cet exercice sont indépendantes les unes des autres.}
\begin{enumerate}
\item Montrer que
\[
\sum_{n\geq 0} e^{-nx} = \frac{1}{1-e^{-x}}
\]
pour tout $x > 0$. En appliquant le théorème de convergence monotone, en déduire que
\[
\int_0^\infty \frac{x e^{-x}}{1-e^{-x}} \dd x = \sum_{n\geq 1} \frac{1}{n^2}\cdot
\]
Connaissez-vous la valeur de cette série ?
\item En utilisant le théorème de convergence dominée, calculer explicitement
\[
\lim_{n\to\infty} \pa{\int_0^\infty \frac{n^2 \sin(x/n)}{(1+nx)(1+x^2)} \dd x}.
\]
\medskip
\item En utilisant le lemme de Fatou, montrer que
\[
\lim_{n\to\infty} \pa{ \int_0^\infty \frac{n}{\sin^2 n + nx^2} \dd x} = \infty.
\]
\end{enumerate}
\end{exo}
\begin{solution}
\begin{enumerate}
\item Comme $x > 0$ on a $e^{-x} \in ]0,1[$ et donc
\[
\sum_{n\geq 0} e^{-nx} = \frac{1}{1-e^{-x}}
\]
par la somme de la série géométrique. On en déduit
\[
\int_0^\infty \frac{x e^{-x}}{1-e^{-x}} \dd x
= \int_0^\infty \sum_{n\geq 0} x e^{-x}e^{-nx} \dd x
= \int_0^\infty \lim_{n\to\infty}\sum_{k= 0}^n x e^{-x}e^{-kx} \dd x
\]
Comme la série est à termes positifs, la suite des sommes partielles est positive est croissante, donc on peut appliquer le théorème de convergence monotone qui entraîne
\[
\int_0^\infty \frac{x e^{-x}}{1-e^{-x}} \dd x
= \lim_{n\to\infty}\int_0^\infty \sum_{k= 0}^n x e^{-(k+1)x} \dd x
= \lim_{n\to\infty} \sum_{k= 0}^n \pa{ \int_0^\infty x e^{-(k+1)x} \dd x}.
\]
Comme
\[
\int_0^\infty x e^{-(k+1)x} \dd x
= \frac{1}{(k+1)^2}\int_0^\infty y e^{-y} dy
= \frac{1}{(k+1)^2}
\]
pour tout $k\geq 0$ en ayant fait le changement de variable $y = (k+1) x$, on trouve finalement
\[
\int_0^\infty \frac{x e^{-x}}{1-e^{-x}} \dd x
= \lim_{n\to\infty} \sum_{k= 0}^n \frac{1}{(k+1)^2}
= \sum_{n\geq 1} \frac{1}{n^2}\cdot
\]
Et oui, nous connaissons bien la valeur de cette série qui est $\pi^2/6$ (problème de Bâle).
\item On a
\[
\frac{n^2 \sin(x/n)}{(1+nx)(1+x^2)}
= \frac{nx}{1+nx}\times\frac{\sin(x/n)}{(x/n)}\times\frac{1}{1+x^2}\to \frac{1}{1+x^2}
\]
quand $n\to\infty$ pour tout $x > 0$ puisque $(\sin z)/z\to 1$ quand $z\to 0$. D'autre part on a $\vert (\sin (x/n))/(x/n)\vert < 1$ et $nx/(1+nx) < 1$ pour tout $x > 0$ et $n\geq 0$, d'où
\[
\abs{\frac{n^2 \sin(x/n)}{(1+nx)(1+x^2)}} < \frac{1}{1+x^2}
\]
pour tout $x > 0$ et $n\geq 0$, et la fonction de droite est intégrable. Par le théorème de convergence dominée, on obtient
\[
\lim_{n\to\infty} \pa{\int_0^\infty \frac{n^2 \sin(x/n)}{(1+nx)(1+x^2)} \dd x}
= \int_0^\infty \frac{1}{1+x^2} \dd x
= \frac{\pi}{2}\cdot
\]
\item On a
\[
\frac{n}{\sin^2 n + nx^2} = \frac{1}{x^2 + (\sin^2 n/n)} \to \frac{1}{x^2}
\]
quand $n\to\infty$ et donc
\[
\liminf_{n\to\infty}\pa{ \frac{n}{\sin^2 n + nx^2}} = \frac{1}{x^2}
\]
pour tout $x > 0$.
Par le lemme de Fatou, on a
\[
\liminf_{n\to\infty} \pa{ \int_0^\infty \frac{n}{\sin^2 n + nx^2} \dd x}
\geq \int_0^\infty \liminf_{n\to\infty} \pa{\frac{n}{\sin^2 n + nx^2}} \dd x
= \int_0^\infty \frac{dx}{x^2}
= \infty
\]
et donc
\[
\lim_{n\to\infty} \pa{ \int_0^\infty \frac{n}{\sin^2 n + nx^2} \dd x}
= \infty.
\]
\end{enumerate}
\end{solution}
% ----------------------------------------------- 4
\begin{exo}
Soit $\suite[n\geq 1]{X_n}$ une suite de variables aléatoires suivant une loi exponentielle de paramètre 1, dont on rappelle que la densité est donnée par $e^{-x}\un{x >0}$ sur $\R$. On suppose que les $X_n$ sont mutuellement indépendantes. On note $M_0 = 0$ et
\[
M_n \coloneqq \max (X_1, \ldots, X_n), \qquad n\geq 1.
\]
\begin{enumerate}
\item Calculer la fonction de répartition $F_n (x) = \PP{M_n \leq x}$ pour tout $n\geq 1$ et $x\in\R$. En déduire $G_n (x) \coloneqq \PP{M_n - \log n \leq x}$ pour tout $n\geq 1$ et $x\in\R$.
\item\label{4b} Calculer la fonction de répartition de $Z_1 = -\log X_1$. En déduire à l'aide de la question précédente et d'un équivalent approprié que
\[
M_n - \log n \darrow -\log X_1
\]
quand $n\to \infty$.
\item En utilisant les fonctions $F_n$ et $F_{n-1}$, montrer que
\[
\EE{M_n} = \EE{M_{n-1}} + \frac{1}{n}
\]
pour tout $n\geq 1$. En déduire que
\[
\EE{M_n} = 1 + \frac{1}{2} + \cdots + \frac{1}{n}\cdot
\]
\indication{On rappelle la formule $\EE{X} = \int_{0}^{\infty}\pa{1-F_X(t)} \dd t - \int_{-\infty}^{0}F_X(t) \dd t$, où $F_{X}$ est la fonction de répartition de $X$.}
\item En utilisant le théorème de convergence dominée et la question \eqref{4b}, montrer que
\[
\EE{M_n} - \log n \to \EE{-\log X_1}
\]
quand $n\to\infty$.\\
\indication{On rappelle que $\log(1-z)\leq -z$ pour tout $z < 1$ et on pourra vérifier que $\log(1-z)\geq -2z$ pour tout $z\in [0,1/2]$.}
\item En utilisant tout ce qui précède, montrer que la suite
\[
u_n = 1 + \frac{1}{2} + \cdots + \frac{1}{n} - \log n
\]
converge quand $n\to\infty$ vers une constante finie, que l'on exprimera sous forme intégrale. Connaissez-vous le nom de cette constante ? Est-elle un nombre rationnel ?
\end{enumerate}
\end{exo}
\begin{solution}
\begin{enumerate}
\item Rappelons que la fonction de répartition de la loi exponentielle est $F(x) = \pa{1 - e^{-x}}_+$, avec la notation $z_+ \coloneqq \max(0,z)$ pour tout $z\in\R$.
Pour tout $n\geq 1$, on trouve
\[
\PP{M_n \leq x}
= \PP{X_1\leq x, \ldots, X_n\leq x}
= \PP{X_1 \leq x} \times\cdots\times\PP{X_n \leq x}
= (1-e^{-x})_+^n.
\]
On en déduit, pour tout $n\geq 1$ et $x\in\R$,
\[
G_n (x)
= \PP{M_n \leq x + \log n}
= \pa{1 - \frac{e^{-x}}{n}}_+^n.
\]
\item\label{s4b} Pour tout $x\in\R$, on calcule
\[
\PP{Z_1 \leq x}
= \PP{\log X_1 \geq -x}
= \PP{X_1 \geq e^{-x}} = e^{-e^{-x}},
\]
car $\PP{X_1 \geq \alpha} = \PP{X_1 > \alpha} = e^{-\alpha}$ pour $\alpha \geq 0$. On en déduit que
\[
G_n (x)
= \pa{1 - \frac{e^{-x}}{n}}_+^n
= \exp\left[n \log\pa{1 - \frac{e^{-x}}{n}}\right]
\]
pour tout $x > -\log n$ et donc, à l'aide de l'équivalent $\log(1-z)\sim z$ quand $z\to 0$ que
\[
\PP{M_n - \log n \leq x} \to e^{-e^{-x}}
= \PP{Z_1 \leq x}
\]
quand $n\to \infty$ pour tout $x\in\R$ puisqu'à partir d'un certain rang on aura $x \geq -\log n\to -\infty$ quand $n\to \infty$. Ceci entraîne par définition que
\[
M_n - \log n \darrow -\log X_1
\]
quand $n\to \infty$.
\item Comme $M_n$ est une variable aléatoire positive, on peut utiliser la formule
\[
\EE{M_n} = \int_0^\infty \PP{M_n > x} dx
= \int_0^\infty (1- F_n(x)) dx
= \int_0^\infty \pa{1 - (1- e^{-x})^n} dx
\]
pour tout $n\geq 1$. Comme
\[
1 - (1- e^{-x})^n
= 1 - (1- e^{-x})^{n-1} + e^{-x} (1- e^{-x})^{n-1}
\]
on obtient
\[
\EE{M_n}
= \EE{M_{n-1}} + \int_0^\infty e^{-x} (1- e^{-x})^{n-1} dx
= \EE{M_{n-1}} + \int_0^1 (1- z)^{n-1} dz
\]
en faisant le changement de variable $z = e^{-x}$, ce qui donne
\[
\EE{M_n}
= \EE{M_{n-1}} + \frac{1}{n}
\]
et donc
\[
\EE{M_n} \;
= 1 + \frac{1}{2} + \cdots + \frac{1}{n}
\]
par itération, puisque $\EE{M_1} =\EE{X_1} = 1$.
\item On a
\[
\EE{M_n} - \log n
= \EE{M_n -\log n}
= \int_0^\infty (1-G_n(x)) dx - \int_{-\infty}^0 G_n(x) dx
\]
par la formule du cours sur l'espérance d'une variable aléatoire réelle.
Par \eqref{s4b} on a $G_n(x)\to G(x) = \PP{-\log X_1 \leq x}$ pour tout $x\in\R$ et sous les hypothèses du théorème de convergence dominée on aura donc
\[
\EE{M_n} - \log n \to \int_0^\infty (1-G(x)) dx - \int_{-\infty}^0 G(x) dx
= \EE{-\log X_1}
\]
quand $n\to\infty$ comme requis. Reste à vérifier les hypothèses du théorème de convergence dominée. On sait que $\log(1-z)\leq -z$ pour tout $z < 1$ et donc
\[
G_n(x)
= \exp\left[n \log\pa{1 - \frac{e^{-x}}{n}}_+\right] \leq e^{-e^{-x}}
\]
pour tout $x\geq 0$, et le terme de droite est une fonction intégrable sur $(-\infty,0)$. On vérifie aussi que $1- z \geq e^{-2z}$ pour tout $z\in [0,1/2]$ puisque la fonction $z\mapsto 1 -z -e^{-2z}$ est nulle en $z=0$, strictement positive en $z=1/2$ et de dérivée $2e^{-2z} -1$ qui est positive puis négative. Ceci entraîne par croissance du logarithme
\[
1- G_n(x)
= 1 - \exp\left[n \log\pa{1 - \frac{e^{-x}}{n}}\right] \leq 1- e^{-2e^{-x}}
\]
pour tout $x\geq 0$ et $n\geq 2$, avec $1- e^{-2e^{-x}} \leq 2e^{-x}$ qui est bien une fonction intégrable sur $]0,\infty[$.
\item En utilisant tout ce qui précède, on voit que
\[
u_n = 1 + \frac{1}{2} + \cdots + \frac{1}{n} - \log n \to \;\EE{-\log X_1}
\]
quand $n\to\infty$. Et par la formule de transfert on a
\[
\EE{-\log X_1}
= \int_0^\infty (-\log x) e^{-x} dx
\]
qui est bien une intégrale convergente en zéro et en l'infini. Il s'agit de la constante $gamma$ d'Euler, dont on ne sait pas si elle est rationnelle ou non (mais on pense que non).
\end{enumerate}
\end{solution}
\end{document}
......@@ -25,6 +25,11 @@
\newcommand{\1}{\mathds{1}}
\newcommand{\un}[1]{\mathbf{1}_{\{#1\}}}
% Macros Thomas
\newcommand*{\deq}{\stackrel{d}{=}}
\newcommand*{\darrow}{\xrightarrow{\kern3pt d\kern4pt}}
\newcommand*{\parrow}{\stackrel{\kern3pt (\P)\kern4pt}{\rightarrow}}
% Macros Kroum
\newcommand*{\suite}[2][n]{\left( #2 \right)_{#1}}% exemple \suite{u_n}, \suite[k>0]{u_k}
\newcommand*{\suiteN}[1]{\suite[n\in\N]{#1}}% exemple \suiteN{u_n}
......@@ -53,6 +58,8 @@
\newcommand{\Bin}[1]{\operatorname{Bin}(#1)}% loi binomiale
\newcommand{\Exp}[1]{\mathcal{E}(#1)}% loi exponentielle
\newcommand{\Uni}[1]{\mathcal{U}(#1)}% loi uniforme
\newcommand{\Nor}[1]{\mathcal{N}(#1)}% loi normale
\newcommand{\LogNor}[1]{\mathcal{L}(#1)}% loi log-normale
% indications
\newcommand{\indication}[1]{\emph{Indication : #1}}
......
......@@ -19,6 +19,7 @@ Dans [ce dépôt](https://gitlab.univ-lille.fr/tzanev/l3m61proba/) vous pouvez t
### Les sujets d'examens et leurs solutions
- DS1 **[[sujet](Exam/M61B_2023-24_DS1-sujet.pdf)]** **[[solutions](Exam/M61B_2023-24_DS1-solutions.pdf)]** [[tex](Exam/M61B_2023-24_DS1.tex)]
- DS2 **[[sujet](Exam/M61B_2023-24_DS2-sujet.pdf)]** **[[solutions](Exam/M61B_2023-24_DS2-solutions.pdf)]** [[tex](Exam/M61B_2023-24_DS2.tex)]
### Source complémentaires
......
......@@ -25,12 +25,18 @@
\newcommand{\1}{\mathds{1}}
\newcommand{\un}[1]{\mathbf{1}_{\{#1\}}}
% Macros Thomas
\newcommand*{\deq}{\stackrel{d}{=}}
\newcommand*{\darrow}{\xrightarrow{\kern3pt d\kern4pt}}
\newcommand*{\parrow}{\stackrel{\kern3pt (\P)\kern4pt}{\rightarrow}}
% Macros Kroum
\newcommand*{\suite}[2][n]{\left( #2 \right)_{#1}}% exemple \suite{u_n}, \suite[k>0]{u_k}
\newcommand*{\suiteN}[1]{\suite[n\in\N]{#1}}% exemple \suiteN{u_n}
\newcommand*{\abs}[1]{\left\lvert{\ifx\hfuzz#1\hfuzz \,\cdot\,\else#1\fi}\right\rvert}% |.|
\newcommand*{\norm}[1]{\left\lVert{\ifx\hfuzz#1\hfuzz \,\cdot\,\else#1\fi}\right\rVert}% norme
\newcommand*{\scalprod}[3][]{#1\langle{#2}\kern1pt #1|{#3}#1\rangle}% exemple \scalprod[\big]{A}{B}
\newcommand*{\ens}[2][]{#1\{ #2 #1\}}% exemple \ens[\big]{A,B,C}
\newcommand*{\ensemble}[3][]{#1\{ #2 \;#1|\; #3 #1\}}% exemple \ensemble[\big]{x^2}{x \in \R}
\newcommand{\iintv}[1]{\ldbrack #1\rdbrack}% intervalle d'entiers
\newcommand{\dd}{\,\mathrm{d}}% le «d» du dx, dt, ...
......@@ -52,6 +58,8 @@
\newcommand{\Bin}[1]{\operatorname{Bin}(#1)}% loi binomiale
\newcommand{\Exp}[1]{\mathcal{E}(#1)}% loi exponentielle
\newcommand{\Uni}[1]{\mathcal{U}(#1)}% loi uniforme
\newcommand{\Nor}[1]{\mathcal{N}(#1)}% loi normale
\newcommand{\LogNor}[1]{\mathcal{L}(#1)}% loi log-normale
% indications
\newcommand{\indication}[1]{\emph{Indication : #1}}
......
0% Loading or .
You are about to add 0 people to the discussion. Proceed with caution.
Please register or to comment